Emergency

¡Supera tus tareas y exámenes ahora con Quizwiz!

A 16-year-old boy is brought to the emergency department after being hit by a car while riding his bike. He is somnolent, opens his eyes to pain, responds to questions with inappropriate words, and uses his left hand to stop the physician attempting to start an IV in his right arm. What is the patient's Glasgow coma score?

10 The Glasgow coma scale (GCS) is a grading of patient responsiveness based on three components: eye opening (1 to 4 points), verbal response (1 to 5 points), and motor response (1 to 6 points). The severity of traumatic brain injury (TBI) is defined using the GCS with mild TBI being defined as a GCS of 14-15, moderate TBI a GCS of 9-13, and severe TBI a GCS 3-8. While other grading tools for mental status exist, the GCS is commonly used due to its interobserver reliability and prognostic value. The Glasgow coma scale has been shown to have prognostic value in traumatic brain injury, spontaneous subarachnoid hemorrhage, and bacterial meningitis. Limitations of the scale include its decreased utility in the setting of drugs, alcohol, sedation, and intubation and its reliance on behavioral response without regard for the underlying pathophysiology. The patient above would receive a score of 2 for eye opening, 3 for verbal response, and 5 for motor response for a total of 10.

Which of the following patients is most likely to sustain a Chance fracture in a motor vehicle collision?

A 32 year old pregnant passenger wearing a lap belt Chance fractures, as seen above, are a variant of the flexion-distraction injury of the spine. These fractures typically occur in the thoracolumbar junction, which spans from T11-L2. Flexion-distraction injuries result from sudden deceleration and most commonly occur in restrained passengers of motor vehicle collisions and people who have fallen from a height. These injuries are most commonly seen in people who wear only lap belts that are improperly positioned above the pelvic bones. This is especially common in pregnant or obese patients. The sudden deceleration associated with a collision leads to forceful flexion at the location of the lap belt, resulting in compression of the anterior and middle columns and tearing of posterior ligamentous support. Flexion-distraction injuries are not typically seen in patients who were wearing three-point restraints at the time of the trauma. A seat belt sign may be seen on physical exam. Radiographic findings in these injuries include posterior vertebral wall fracture, increased height of the posterior vertebra, and fanning of the spinous processes. The Chance fracture also involves anterior vertebral compression and significant distraction of the middle and posterior ligaments. Chance fractures are often misdiagnosed as isolated compression fractures when viewed in the axial view during computed tomographic imaging, and sagittal reconstruction is often necessary for proper diagnosis. Chance fractures are commonly associated with intra-abdominal injuries and a high index of suspicion should be maintained for bowel perforation or solid organ damage in patients with Chance fractures.

Which of the following is the treatment of choice in preventing acute mountain sickness?

Acetazolamide Acute mountain sickness (AMS) is characterized by symptoms similar to those with a mild viral illness or "hangover" including headache, nausea, vomiting, fatigue, dizziness, and difficulty sleeping. Symptoms typically occur within hours of reaching a high altitude (generally > 8,000 feet) and peak in 24-48 hours. Slow ascent to allow time for acclimatization is the best method of prevention. When that is not possible or when there is a previous history of acute mountain sickness with ascent, use of acetazolamide (125-250 mg twice daily from one day prior to ascent and continuing for 48 hours after reaching altitude) has been shown to prevent the majority of symptoms. Acetazolamide is a carbonic anhydrase inhibitor that induces a renal bicarbonate diuresis resulting in a metabolic acidosis which thereby increases ventilation and arterial oxygenation. The bottom graphic demonstrates the decline in partial pressure of inspired oxygen compared to its value at sea level.

A 35-year-old man presents with headache, nausea, and lightheadedness. The headache started 1 day after a flight from Boston to Denver. He describes the headache as bifrontal and throbbing. The patient has normal vital signs and his neurologic exam is normal. The patient's presentation is consistent with which of the following disorders?

Acute mountain sickness The patient has acute mountain sickness (AMS). In order to diagnose AMS, the patient must have a recent gain in altitude which occurred at least several hours ago, a headache, and one of the following additional symptoms: gastrointestinal upset, generalized weakness, fatigue, dizziness, lightheadedness, or insomnia. The headache is generally bitemporal and throbbing. Headaches are generally worse at night and when moving to an upright position. The pathophysiology of AMS is not fully understood, but is thought to be caused by hypoxia which leads to intracranial vasodilation and cerebral edema. Insomnia is caused by nocturnal, periodic breathing. The cornerstone of management of AMS is to halt ascent until symptoms improve. Headaches can be treated with acetaminophen or ibuprofen. Supplemental oxygen may also reduce symptoms. Acetazolamide can be used to stimulate breathing and can be helpful for insomnia. It is a carbonic anhydrase inhibitor which causes a renal bicarbonate diuresis. This causes a metabolic acidosis which stimulates increased ventilation and thereby improves arterial oxygenation. If symptoms do not improve or worsen with conservative management, the patient should descend to lower altitudes.

A 35-year-old woman with a prior history of suicidal ideations is brought to the Emergency Department approximately 10 hours after ingesting an unknown quantity of acetaminophen and ibuprofen. She is complaining of nausea and abdominal pain. Her blood pressure is 150/80 mm Hg, heart rate is 90, respiratory rate is 18, and temperature is 36.8°C. Physical examination reveals diffuse abdominal pain. What is the most appropriate next step in management?

Administer N-acetylcysteine Acetaminophen ingestion remains one of the leading causes of hospital admissions due to poisoning in the United States and in the world. Risk of severe hepatotoxicity increases if the patient is not treated with N-acetylcysteine (NAC) within the first 8 hours after ingestion. Severe hepatotoxicity may develop and lead to multiorgan failure and death if the poisoning is not treated appropriately. If patient presents within the first 4 hours after ingestion, acetaminophen and transaminases must be obtained at 4 hours after the ingestion and Rumack-Matthew nomogram can then be used to determine if treatment is needed. If the patient presents after 8 hours from the time of the ingestion, a loading dose of NAC at the initial dose of 140 mg/kg should be administered without delay. Acetaminophen level as well as transaminases must be determined, but therapy should not be delayed while awaiting the results of the laboratory studies. NAC is available in IV and PO formulations, with compatible bioavailability. Intravenous NAC must be administered in cases of fulminant liver failure and pregnancy. All intentional ingestions require a psychiatric consultation once the patient is medically stable.

A 13-year-old boy is brought to the emergency room by his mother after a suicide attempt. She reports that he swallowed a bottle of pills but is unsure of what he ingested. His vital signs are BP 90/60, HR 135, RR 16, and T 100.3°F. On exam, the patient is somnolent, his pupils are dilated, mucous membranes are dry, and skin is warm and flushed. Which of the following substance did this patient most likely ingest?

Amitriptyline Amitriptyline is a tricyclic antidepressant (TCA) and the most frequent cause of poisoning in this class of drugs. Overdose can lead to a variety of anti-cholinergic symptoms, including tachycardia, dry mouth, mydriasis, hyperreflexia, warm flushed dry skin, gastrointestinal complaints, urinary retention, and confusion or agitation. Severe overdose can cause life-threatening symptoms such as coma, seizure, dysrhythmia, and cardiac arrest. Initial management may include gastric decontamination with activated charcoal but is otherwise supportive, depending on the presenting symptoms (intravenous fluids, cardiac agents, respiratory support).

A five-year-old girl is brought by her father to the clinic because of a cat bite. Yesterday, she was playing with a neighbor's cat that suddenly bit the girl's left hand. Her wound was immediately cleaned. The following day, the father noted increased swelling and redness of the girl's left hand. On physical examination, she has normal vital signs, and on the left hand are two puncture wounds with a surrounding 1 cm diameter of erythema and swelling. Which of the following is the best treatment for a suspected Pasteurella sp. soft tissue infection?

Amoxicillin-clavulanate The girl has a soft tissue infection from a cat bite. Clinical soft tissue infections with Pasteurella multocida usually occur after cat bites, cat scratches, or dog bites but may also occur following cat or dog licks of non-intact skin. P. multocida wound infections characteristically have a very rapid development of an intense inflammatory response. Most patients develop symptoms within 24 hours of the initial injury, and as early as three hours after a cat bite. Pain and swelling are prominent. Purulent drainage is noted in about 40 pecent of patients, lymphangitis in about 20 percent, and regional adenopathy in 10 percent. Cellulitis often occurs within 24 to 48 hours. Necrotizing fasciitis may occur. Pasteurella is the first organism to consider in any patient who presents with a soft tissue infection following cat scratches or cat or dog bites or licks. The diagnosis of P. multocida infection is made by isolation of the organism in culture. Pasteurella sp., including P. multocida, are usually susceptible to a number of antibiotics, including amoxicillin-clavulanate, piperacillin-tazobactam, doxycycline, fluoroquinolones, advanced cephalosporins, and carbapenems.

A 24-year-old woman presents to the Emergency Department after a high-speed motor vehicle collision. Her GCS is 14. On neurological examination, she has intact fine touch, but decreased strength and loss of pain and temperature sensation below the T10 level. What is the most likely diagnosis?

Anterior cord syndrome Injury to the anterior two-thirds of the spinal column results in anterior cord syndrome which is characterized by bilateral loss of motor function and pain and temperature sensation below the level of injury. The dorsal column is usually spared so vibration sense and proprioception are preserved. Anterior cord syndrome occurs in cases of disruption or injury to the anterior spinal artery which can result from protrusion of bone fragments from a traumatic injury to the vertebrae, disc herniation, or following aortic surgery. It can also result from a flexion/compression injury to the cord. Most improvement in function occurs in the first 24 hours. Anterior cord syndrome carries the worst prognosis of all the incomplete spinal cord syndromes, with less than 20% regaining any degree of muscle function.

A 17-year-old baseball player presents to the emergency department complaining of sudden left shoulder pain. The patient states he heard his shoulder "pop" and felt pain as he was extending his arm back to throw a fastball. On examination, the patient is holding his left shoulder in abduction and external rotation with loss of the deltoid contour. There is limited and painful range of motion. Which of the following is the most likely diagnosis?

Anterior glenohumeral dislocation of the left shoulder The patient suffered an anterior glenohumeral dislocation of his left shoulder. The shoulder is the most commonly dislocated joint in the body. Dislocations can occur from a trauma (95%) or from laxity of the ligaments (5%). The glenohumeral ligaments, the rotator cuff muscles, and the joint capsule or labrum stabilizes the shoulder. The shoulder joint is often injured during a fall on an outstretched arm, engaging in sports activities, a direct blow to the joint, or grand mal seizures. Patients often report hearing or feeling a pop during the incident and present with significant pain, limited range of motion, swelling, or deformity. Anterior shoulder dislocations are the most common type; however, they can occur posteriorly and inferiorly. Anterior dislocations result from a force applied to the shoulder while the arm is abducted and externally rotated such as while throwing a ball. On examination, the arm is held in abduction and external rotation (down adjacent to the body) with a loss of the deltoid contour when compared to the other shoulder, and the humeral head can often be palpated anteriorly beneath the clavicle. The patient will have very limited range of motion and be unable to touch the opposite shoulder. Obtaining pre-reduction and post-reduction radiographs and neurovascular examinations are imperative to successful management. Complications of shoulder dislocations include fractures (e.g. Hill-Sachs lesion), soft tissue injury (e.g. Bankart lesion), nerve injury (e.g. axillary nerve), or vascular injury (e.g. axillary artery).

A 66-year-old man presents after a landscaping injury. He was pruning bushes with motorized shears when he slipped and cut his right thigh. He put a towel on the wound and was able to walk to the Emergency Department. He has no other injuries and his vital signs are within normal limits. He takes warfarin for atrial fibrillation. When you examine his wound, you note a seven-centimeter simple laceration with blood continuously oozing from the defect. Which of the following is the appropriate next step?

Applying direct pressure to the wound Bleeding is a leading cause of mortality in patients suffering from trauma. Rapid identification and control of external hemorrhage is part of the primary trauma survey. Even with normal vital signs, a significant volume of blood can be lost. Control of the bleeding can prevent further blood loss. Arterial bleeding is under pressure and will spurt, while venous bleeding will generally be a continuous slow ooze. Direct pressure to the wound is an effective technique to gain early control of a bleeding injury. Pressure compresses the vasculature and gives time for a clot to form at the site of the injury.

A 64-year-old woman is brought to the emergency room by ambulance after being the restrained driver in a motor vehicle collision. She fell asleep at the wheel, crossed the highway median, and hit another car head-on at 60 miles per hour. There was a fatality in the other vehicle. On examination, the patient is alert and screaming in pain. Her exam is notable for absent shoulder shrug, 0/5 strength of the upper extremities and 5/5 strength of the lower extremities. Her X-ray reveals anterior displacement of the occiput relative to the atlas. Which of the following is the most likely diagnosis?

Atlanto-occipital dissociation Atlanto-occipital dislocation, commonly referred to as internal decapitation, is a highly unstable and severe ligamentous injury at the craniocervical junction. It is often immediately fatal, but, more recently, more patients have been surviving due to advances in prehospital medicine. Atlanto-occipital dislocation can be caused by a variety of different traumatic mechanisms that transmit excessive force to the craniocervical junction leading to ligamentous disruption. The most common mechanism of injury is high-speed motor vehicle collision. The classic presentation is cruciate paralysis, with paralysis of the upper extremities and sparing of the lower extremities. However, atlanto-occipital dislocation can present with a wide spectrum of symptoms, ranging from isolated neck pain to lower cranial nerve deficits, unilateral or bilateral weakness, and quadriplegia. Spinal cord injury is relatively rare but when present is often immediately fatal due to brainstem injury. Diagnosis can be made by computed tomography or X-ray and is based on abnormalities in the measurements of the basion-dental interval and the basion-atlantal interval. Management of atlanto-occipital dislocation in the field and the emergency department includes confirmation of a stable airway, hemodynamic stabilization, and application of rigid cervical collar. Definitive management is through halo placement.

A patient presents with symptomatic bradycardia following an overdose of metoprolol. After securing the airway, the most appropriate next step is administration of which of the following?

Atropine Atropine is used to treat symptomatic bradycardia. Metoprolol is a beta-adrenergic antagonist. Beta blockers are categorized according to their cardioselectivity. Cardioselective beta-blockers preferentially inhibit beta-1 receptors, resulting in depressed myocardial contractility, decreased automaticity of pacemaker cells, and decreased conduction velocity through the atrioventricular node. Patients with overdoses of beta-blockers usually become symptomatic within two to six hours after ingestion. Bradycardia and hypotension are the most common effects. Altered mental status, seizures, hypoglycemia, and bronchospasm are other potential effects. Hypoglycemia is more commonly seen in children. If beta blocker overdose is suspected, an ECG, fingerstick glucose, and electrolyte panel should be obtained. Airway management and advanced cardiac life support (ACLS) should be provided, followed by establishing intravenous (IV) access and continuous cardiac monitoring. Treat hypotension with IV boluses of isotonic fluid and hypoglycemia with boluses of 50 percent dextrose in water. IV glucagon, vasopressors, and calcium salts can be used depending upon the severity of the overdose. Glucagon is considered to be antidote for beta blocker overdose despite limited evidence. Glucagon may be effective initially for a brief period, but will usually become ineffective due to tachyphylaxis and has little effect on mean arterial pressure. High dose insulin therapy, however, is now the mainstay in treatment of beta blocker toxicity.

A 36-year-old man presents after being bitten by "something". He felt a pinprick sensation on his forearm while cleaning out his garage followed by worsening pain over the next sixty minutes that spread to involve his entire arm. Shortly after, he began having severe cramping of his chest and abdomen. What was he likely bitten by?

Black widow spider Black widow spiders, Latrodectus mactans, are found throughout the United States. The female is twice as large as the male and is the only one able to envenomate humans. The spider is glossy black and has a bright red marking, either two spots or an hourglass figure, on the abdomen. It is not typically aggressive unless protecting its eggs. The venom is highly potent and results in release of acetylcholine resulting in neuromuscular symptoms and norepinephrine resulting in cardiovascular symptoms. Patients will typically recall a pinprick sensation at the site of the bite followed by mild erythema and swelling. Within minutes to an hour, the pain increases and spreads to the entire body. Severe cramping pain occurs in the chest, abdomen, and back and can mimic an acute abdomen. Systemic symptoms include nausea, vomiting, headache, and dizziness. Hypertension and tachycardia are common. Symptoms typically last several hours to days, with resolution in 2-3 days. Young children and patients with preexisting hypertension and cardiovascular or cerebrovascular disease are at highest risk for complications.

A 19-year-old man presents to the ED with facial and mouth pain after being assaulted with a club. On physical exam, you note significant swelling over the left inferior and lateral face and misalignment of the inferior premolars. Which of the following is the most common anatomic location of this traumatic injury?

Body of mandible This patient has signs and symptoms consistent with a mandibular fracture. The mandible is the second most commonly fractured facial bone behind the nasal bone. The most common fracture location is the body of the mandible, which is more likely to show signs of obvious tooth misalignment on exam. Other signs and symptoms of a mandibular fracture include trismus, malocclusion of the bite, intraoral lacerations, and missing teeth. The tongue blade test is a way of detecting even subtle mandibular fractures. The patient bites the blade while the examiner attempts to break it. Inability to stabilize the blade is an indication that the patient may have a mandibular fracture and should undergo imaging. Plain mandibular X-rays and panoramic radiographs are screening tests for mandibular fractures; however, CT of the facial bones is the gold diagnostic standard. Management is variable depending upon location and whether the fracture is open or closed. Open fractures require IV antibiotics, such as penicillin or clindamycin, and hospital admission. Closed fractures may be discharged with dental follow-up. The patient should be given adequate oral analgesics and instructed to observe a soft diet and chew on the non-fractured side. Many fractures will require open reduction and internal fixation, but do not require admission initially if not open fractures.

A seven-year-old boy is involved in a rollover motor vehicle collision. He arrives intubated by the paramedic service. On exam, you note him to be flexing both upper extremities. Which of the following is the most sensitive indicator of herniation?

Bradycardia Cushing's triad is usually a pre-terminal event seen in patients with increased intracranial pressure and cerebral herniation through the foramen magnum. It is associated with decreased level of alertness, hypertension, bradycardia, and irregular respirations. Bradycardia is the first sign and is therefore the most sensitive indicator.

A 70-year-old man presents to the ED after a fall down twelve stairs at home. He complains of a headache. There is a large parietal scalp hematoma. His non-contrast computed tomography scan of the head is shown above. Which of the following is the most likely source of bleeding? Picture: black in central R and lighter on lateral L

Bridging veins This patient's imaging and physical examination are consistent with a traumatic acute subdural hematoma. A subdural hematoma is a collection of blood between the dura and the arachnoid mater. Subdural hematomas occur when there is rupture of the bridging veins from movement of the brain relative to the skull, such as that seen with acceleration-deceleration injuries. Subdural hematomas occur most commonly in individuals with brain atrophy, such as alcoholics and elderly patients. This results in blood filling the potential space between the dura and arachnoid. Subdural hematomas may be acute, subacute, or chronic. Acute subdural hematomas are characteristically seen after a traumatic head injury. Patients typically present with headache, mental status changes, seizures, or focal deficits. Subdural hematoma is diagnosed on non-contrast CT of the head and appears as a crescent-shaped hematoma that may cross suture lines. Management of subdural hematomas includes neurosurgical consultation for possible surgical evacuation; however, small subdural hematomas may be closely observed with serial non-contrast CTs of the head. Indications for emergent surgery in a patient with a subdural hematoma include neurologic deterioration or > 5 mm midline shift on CT.

A 34-year-old man presents to the ED with vomiting after exposure to pesticides while working as a farmer. Which of the following would you expect to find on physical exam?

Bronchorrhea This patient is exhibiting signs and symptoms consistent with cholinergic toxicity secondary to organophosphate poisoning. Signs and symptoms of organophosphate poisoning relate to excess acetylcholine at the nicotinic and muscarinic receptors. This produces the cholinergic toxidrome due to high postsynaptic parasympathetic activity, resulting in the classic SLUDGE mnemonic: Salivation, Lacrimation, Urination, Diarrhea, GI cramps, and Emesis. Patients will also demonstrate diaphoresis, pupillary miosis, severe bronchorrhea, bradycardia, muscular fasciculations, paralysis, agitation, seizures, or even coma. Management of organophosphate poisoning should begin with decontamination. Organophosphates bind tightly to acetylcholinesterase preventing the breakdown of acetylcholine. There are two keys to definitive treatment in patients with organophosphate poisoning. The first is to temporize the life-threatening signs and symptoms of cholinergic toxicity. Atropine is a competitive inhibitor of acetylcholine at muscarinic receptors. The goal of treatment with atropine is to titrate to the drying of bronchial secretions. Pralidoxime (2-PAM) is the definitive antidote to organophosphate poisoning. 2-PAM forms a complex with the bound acetylcholinesterase enzyme to cause the release of the organophosphate from the enzyme. This results in regeneration of its ability to metabolize acetylcholine. Pralidoxime is only effective if given early as the organophosphate-acetylcholinesterase complex matures quickly resulting in the inability of 2-PAM to cause regeneration of the enzyme.

An 18-year-old man presents after being stabbed in the back. His vital signs are stable and his FAST examination is negative. Physical examination demonstrates a stab wound to the lower back located just to the left of the midline. The patient is unable to move his left leg and reports that his right leg is numb. Which of the following describes his syndrome?

Brown-Sequard syndrome Brown-Séquard syndrome is a hemisection of the spinal cord most commonly occurring after a penetrating injury. In this syndrome, patients have proprioception and vibratory sensory loss on the ipsilateral side of the injury as well as loss of motor function on the same side. The contralateral side loses pain and temperature sensation since these fibers cross the spinal cord at the level of their nerve roots.

You are called to the emergency department to see a five-year-old boy for ingestion. He was playing in the garage when his mother found him with an opened bottle of antifreeze. The liquid was all over his mouth and clothes. He had two episodes of vomiting and fell asleep on the way to the hospital. On physical examination, you note tachycardia. Which of the following laboratory abnormalities would you expect in this type of ingestion?

Calcium oxalate crystals in urine Ethylene glycol is commonly found in antifreeze. Unintentional ingestion is the most common exposure in children, and even small-volume ingestions of concentrated products have the potential for severe toxicity. Early symptoms begin with nausea, vomiting, CNS depression. Late manifestations include anion gap metabolic acidosis, hypocalcemia, and kidney failure (secondary to deposition of calcium oxalate crystals in the renal tubules). It can be diagnosed by ethylene glycol blood concentrations. Osmolar gap may also be used as a surrogate marker. Examination of the urine with a Wood lamp is neither sensitive nor specific for ethylene glycol ingestion. Calcium oxalate crystals can be seen on urine microscopy. The evaluation of patients should include electrolytes (including calcium), acid-base status, kidney function, and ECG. Gastric decontamination is generally not of value because ethylene glycol is rapidly absorbed. The classic antidote for ethylene glycol poisoning was ethanol. However ethanol has been replaced by fomepizole, a potent competitive inhibitor of alcohol dehydrogenase. This is because of the ease of administration, lack of CNS and metabolic effects, and overall excellent patient tolerability profile of fomepizole.

A 17-year-old high school football player is seen in your clinic with neck pain after practice. He reports the pain started after he tackled a teammate and was associated with tingling in both arms. What is next best step in management?

Cervical spine immobilization This patient is reporting neck pain and neurological symptoms after football practice, which is concerning for a cervical spinal cord injury. Initially, the patient's spine should be immobilized while simultaneously assessing the airway, breathing, and circulation (ABCs). If the injury occurs on the football field, the helmet and shoulder pads should not be removed to ensure spine stabilization. Initial immobilization should be done by stabilizing the head and neck with two hands. If the player is not supine, he should be log-rolled into supine position for the initial assessment. If there is airway compromise, the face mask should be removed. In a clinically stable patient immobilization is done by placing the patient in supine position and placing the neck in a rigid cervical collar. The most common mechanism of injury in adolescents is via axial loading, which can cause a cervical fracture of C1 (Jefferson fracture). This is most likely to occur in football, when a player runs headfirst into another player, and diving, when a diver's head hits the bottom of the pool.

A 1-year-old boy presents unconscious to the emergency department with multisystem trauma after he was a passenger in a motor vehicle accident. He is placed a cervical collar. Which of the following, can definitively clear the cervical spine?

Cervical spine magnetic resonance imaging A cervical spine MRI scan is the definitive way to diagnose and clear a cervical spine injury on patients of all ages. This diagnostic study allows for clear visualization of the ligaments, intervertebral disc spaces, spinal cord, and bones. When a patient presents with a cervical injury, protecting the area with a cervical collar is important in order to maintain spinal alignment and prevent further injury. If a patient is young, unconscious, or unable to communicate, it is difficult to clinically clear the cervical spine. Obtaining a cervical spine X-ray should be the initial course of action to determine whether or not there is movement between the bones. An X-ray, however, is unable to visualize the spinal cord and ligaments. A CT scan of the cervical spine would be the next ideal imaging technique, but this is best to further investigate the bones and not the ligaments or spinal cord. To determine injury to the spinal cord or ligaments, determine the presence of hemorrhage or edema, and to evaluate the intervertebral disc spaces, an MRI of the cervical spine provides the most amount of information and therefore is the best tool to definitively clear the cervical spine before cervical collar removal.

A 26-year-old man presents to the emergency department after being the restrained driver in a motor vehicle collision. He is alert and interactive and complains of moderate abdominal pain. His vital signs are T 37.3°C, HR 99, BP 125/84, RR 19. His physical examination is significant for anterior abdominal wall bruising and mild diffuse tenderness to palpation of the abdomen. A bedside ultrasound assessment reveals the image above. Which of the following is the most appropriate next step in the management of this patient? Positive fast

Computed tomographic imaging of abdomen and pelvis Motor vehicle collisions are the most common cause of blunt abdominal trauma. Injuries vary based on the mechanism and force of the trauma and can include damage to both intra- and extra-abdominal structures. The most commonly damaged intra-abdominal structure in blunt abdominal trauma is the spleen. Intra-abdominal injury may not initially be obvious on physical exam and a normal appearing abdomen does not exclude serious intra-abdominal injury. Abdominal pain, distention, and abdominal wall bruising should raise suspicion for intra-abdominal injury. The focused assessment with sonography in trauma (FAST) is one of the primary diagnostic modalities for diagnosing serious intra-abdominal injury. It includes sonographic examination for free fluid in the pericardium, the hepatorenal and splenorenal recess, and the paracolic gutters and can be extended to include an examination of the thorax. The greatest benefit of FAST is the identification of intraperitoneal free fluid in the hemodynamically unstable patient without an obvious bleeding source. FAST can also be used as an initial assessment in the hemodynamically stable patient. FAST is quick to perform and is preferred to diagnostic peritoneal lavage due to its non-invasive nature. FAST cannot distinguish between blood and urine or ascites, cannot detect subcapsular injuries, and is insensitive at detecting bowel injury. Stable patients with a positive FAST should undergo a computed tomographic (CT) scan of the abdomen to reveal the source of the fluid.

A 2-year-old girl presents to the emergency department at 6 pm with bloody vomiting, diarrhea and abdominal pain. Her pregnant mother reports that the patient's grandfather was watching the patient in the afternoon while the mother was shopping. The mother returned home at around 5pm to find her daughter vomiting and with diarrhea and abdominal pain. An abdominal X-ray shows radiopaque particles in the stomach. After fluid resuscitation, what is the next best step in management?

Deferoxamine administration Deferoxamine is a chelating agent that binds iron and is indicated for persistent gastrointestinal symptoms, if pills are visible on radiographs, if the serum iron level is greater than 500 µg/dL, or if an estimated dose greater than 60 mg/kg of elemental iron was ingested. Iron poisoning is a common toxicologic emergency in pediatrics because of the availability of iron tablets and the often candy-like appearance. In the above case, the mother was likely taking iron supplementation which the child ingested. Toxicity from iron can occur at doses as low as 10 to 20 mg/kg of elemental iron and the severity of poisoning is based on the amount of elemental iron ingested. There are typically 5 stages of iron toxicity. Stage 1 occurs within the first 6 hours of ingestion and includes hemorrhagic vomiting, diarrhea and abdominal pain secondary to mucosal injury. Blood loss can be so severe that it leads to shock. Stage 2 occurs 6 to 24 hours after ingestion and can include signs of clinical improvement; however, severe cases progress to systemic injury presenting as lethargy, tachycardia and tachypnea. Stage 3 occurs 6 to 72 hours after ingestion and consists of multisystem shock, including metabolic acidosis, coagulopathy, seizures, altered mental status and liver damage. Stages 4 and 5 occur in the following days and weeks and can include further liver damage and scarring of the GI tract. Although not required to make the diagnosis, an abdominal xray can be helpful in confirming radiopaque tablets or particles; however, the absence of these findings cannot exclude iron toxicity.

A 72-year-old man presents to the Emergency Department with chest pain. During triage, he collapses and nursing staff cannot feel his pulse. The patient is taken to a treatment room where cardiopulmonary resuscitation is initiated and the above cardiac rhythm is noted. What is the next best step?

Defibrillation den cardiac death is a leading cause of mortality. It occurs in a bimodal age distribution: infancy and individuals > 45 years or age. There is a slight predominance in men. In a sudden episode of cardiac death, the initial rhythm is most likely ventricular fibrillation, or ventricular tachycardia that degenerates into ventricular fibrillation. In a witnessed cardiac arrest, if the initial rhythm is amenable to defibrillation, a shock should be administered as soon as possible. Defibrillation is the best chance of recovering a perfusing rhythm from a nonperfusing rhythm. The longer a heart is in a nonperfusing rhythm, the less likely it is to respond to defibrillation and cardiac life support.

A four-year-old girl is seen at the ED because of possible ingestion. She was unattended for several minutes when she went through her grandmother's bag containing bottles of bethanecol, clonidine, diphenhydramine, and fluoxetine. Upon arrival at the ED, her examination reveals temperature of 38°C, heart rate of 120, respiratory rate of 16, blood pressure 85/56, pupils 5 mm equal and reactive and dry skin. Which of the following is the most likely medication that caused the girl's findings?

Diphenhydramine The girl has clinical findings of anticholinergic poisoning that consists of hyperthermia, tachycardia, mydriasis, and dry skin. Other symptoms include decreased bowel sounds, urinary retention, disorientation, bizarre behavior, paranoia, delirium, visual hallucinations, and, in severe cases, seizures. The classic description of anticholinergic intoxication is known as: "red as a beet," "dry as a bone," "hot as a hare," "blind as a bat," and "mad as a hatter." Diphenhydramine is a medication with anticholinergic effects, and this is the medication the girl in the vignette most likely ingested. Examples of classes of medications with anticholinergic properties include antihistamines, tricyclic antidepressants, sleep aids and cold preparations. Many plants, such as jimson weed (Datura stramonium) and deadly nightshade (Atropa belladonna), may also produce anticholinergic toxicity. Initial management begins with stabilization of the airway, breathing, and circulation. Most patients do well with supportive care alone, but some may benefit from antidotal therapy with physostigmine.

A 69-year-old man with a history of atrial fibrillation had a syncopal episode while on vacation in India and struck his head on the stairs. He was transported to the local clinic to be evaluated. His wife accompanied him and she informs the provider that her husband is on a "blood thinner" but she is not sure which one. The patient is alert but is not oriented to time or place. You suspect a subdural hematoma but there is no access to a CT scanner at this time. His vital signs are within normal limits. While awaiting transportation to a facility with a CT scan, which of the following labs should be ordered?

Elderly patients on anticoagulation therapy are at high risk of subdural hematomas when they fall. In general, elderly patients are at risk of falls because they tend to have balance difficulties and decreased vision in older age. A non-contrast CT of the head is the gold diagnostic standard for diagnosing a subdural hematoma. A subdural hematoma usually appears as a hyper dense, crescent-shape-mass between the skull and the brain. A coagulation panel is necessary in the workup of a suspected subdural hematoma because individuals on anticoagulants or who are alcoholics may have an associated coagulopathy placing them at higher risk for a subdural hematoma. In addition, if INR levels are found to be elevated, reversal agents (e.g. vitamin K and fresh frozen plasma can be administered). Therefore, all patients with a head injury should have at least a basic coagulation panel (INR, PT, aPTT, and platelet count). Fresh frozen plasma or platelets should be given as needed.

A 3-year-old boy is eating with his brother in the other room when his mother hears coughing and choking. His brother reports that he had just eaten a handful of peanuts before the symptoms began. In the Emergency Room, the child has moderate respiratory distress with a respiratory rate of 30 breaths per minute and a saturation of 93% on room air. He is placed on oxygen by facemask with improvement in saturations to 96%. Faint expiratory wheezes are noted over the left lobe. The remainder of the examination is unremarkable. A chest radiograph is normal. Which of the following is indicated?

Emergent bronchoscopy The onset of acute respiratory distress following consumption of peanuts should prompt consideration of both foreign body aspiration and anaphylaxis. This child's focal wheezing, especially located over the left lobe, make foreign body aspiration most likely. Notably, chest radiography visualizes only ten percent of foreign bodies, as the majority are radiolucent. Chest radiography may also reveal focal air trapping, mediastinal shift with expiration, or focal atelectasis. However, these findings are not sufficiently sensitive to rule out a foreign body aspiration. In fact, chest radiography may be normal in up to two thirds of cases. If foreign body aspiration is suspected based on history and physical examination, emergent bronchoscopy should be performed. In fact, delayed bronchoscopy has been shown to increase morbidity and mortality in children with foreign body aspiration.

A 19-year-old man presents to the emergency department after a motor vehicle collision. He is unconscious with a unilaterally dilated pupil. What is the most appropriate initial intervention?

Endotracheal intubation Endotracheal intubation should be the initial clinical intervention when a patient presents unconscious with a suspected head injury in order to secure the airway. The primary survey in an emergency is the initial rapid identification, assessment, and management of life-threatening injuries and illnesses. This survey consists of rapid assessment of the airway, breathing, and circulation. The primary survey continues with assessing a patient's disability using screening tools such as the Glasgow Coma Scale followed by fully exposing the patient in order to perform a thorough inspection of the body. Once the primary survey is completed and life-threatening injuries are stabilized, the clinician can move onto the secondary survey which is a rapid identification and assessment of injuries that have the potential to become unstable. For instance, controlling bleeding or evaluating pelvic instability. When a patient initially presents to the emergency department, assessing the airway is the most important intervention and therefore endotracheal intubation should be the primary clinical intervention in an unconscious patient.

When prescribing metronidazole which of the following should patients be specifically instructed to avoid?

Ethanol Metronidazole is used in the treatment of various parasitic and bacterial infections. It is both effective and well-tolerated. However, patients should be specifically instructed to avoid alcohol intake, as even a small amount of alcohol ingestion during metronidazole therapy may result in intense vomiting, also known as a disulfiram-like reaction. Disulfiram is a medication used to treat alcoholism by inhibiting the complete breakdown of ethanol. When acetaldehyde, a byproduct of ethanol metabolism, accumulates in the blood, it causes nausea, vomiting, flushing, palpitations, headache, and lowered blood pressure. This constellation of symptoms constitutes the disulfiram reaction.

A 21-year-old woman presents to the Emergency Department after an intentional ingestion. On physical examination, she is lethargic, flushed, hot to the touch, and her pupils are dilated. Vital signs include BP 130/90 mm Hg, HR 130 beats/minute, and RR 22 breaths/minute. After several minutes, she has a generalized tonic clonic seizure. What is the mechanism by which this class of medications causes cardiac toxicity?

Fast inward sodium channel blockade This patient is exhibiting signs and symptoms of a tricyclic antidepressant (TCA) overdose. TCAs act by fast inward sodium channel blockade, causing QRS complex widening (> 100 ms) on ECG. TCAs also have anticholinergic effects, which lead to a presentation of pupillary dilation, dry, hot, flushed skin, tachycardia, decreased or absent bowel sounds, and urinary retention. In addition to severe cardiac effects, TCAs can also cause rapid deterioration of mental status and seizures. The most important first step in diagnosis is to perform an electrocardiogram to evaluate for widening of the QRS complex. Another hallmark finding of TCA toxicity on ECG is a dominant terminal R wave in lead aVR. Management of TCA toxicity includes administration of sodium bicarbonate to combat the sodium channel blockade if there is widening of the QRS complex on ECG or telemetry. Sodium bicarbonate boluses should be administered, 50 mEq IV at a time, until the QRS complex narrows. A continuous infusion of 150 mEq of sodium bicarbonate in 1 L D5W at a rate of two to three times maintenance should be initiated at that time. Potassium levels will need to be monitored closely while sodium bicarbonate is infusing. Lorazepam or diazepam should be administered for seizures. Intubation may be required for airway protection given the potential for neurologic deterioration. Any patient with a symptomatic TCA overdose should be admitted for telemetry monitoring and frequent neurologic checks.

A 2-year-old unvaccinated boy is rushed to the Emergency Department with concerns for respiratory distress. His mother reports that he awoke in his usual state of health and was playing alone in the playroom when mom heard him have a coughing fit. Since that time, he has been choking and coughing, has had inspiratory stridor, and developed significant increased work of breathing. What is the most likely etiology of his stridor?

Foreign body aspiration The child's sudden onset of respiratory distress in an unobserved setting is suggestive of a foreign body aspiration. The child's inspiratory stridor is indicative of an upper airway obstruction, and he most likely has a laryngotracheal foreign body aspiration. Aspiration into the bronchi, and especially the right mainstem bronchus, is significantly more common than laryngotracheal aspiration. However, objects with sharp or irregular edges are more likely to become lodged in the upper airway. Children with laryngotracheal foreign body aspiration require immediate medical attention for airway stabilization and removal of the foreign body. Notably, if a child is able to cough, the airway obstruction is partial. A blind finger sweep of the mouth is not recommended, as this may lead to dislodgment of the foreign body and subsequent complete airway obstruction. Radiographic evaluation of the neck is recommended, but only approximately ten percent of foreign bodies are radio-opaque. Instead the radiograph may show subglottic swelling. However, radiographs are insufficient to rule out a foreign body. If the diagnosis is suspected, it should be further evaluated by bronchoscopy.

A three-year-old boy is brought to the emergency department due to acetaminophen ingestion. About four hours prior, he was found by his mother with an open bottle of acetaminophen and several tablets scattered on the floor. The mother was not sure how many tablets were missing. The boy had one episode of vomiting at home. He is asymptomatic upon arrival at the emergency room. On examination, the boy is active with normal vital signs and physical examination. Which of the following is the most appropriate next step in management?

Get acetaminophen level four hours after ingestion Acetaminophen is commonly present in the home, which can be unintentionally ingested by young children. The initial signs of acetaminophen toxicity are nonspecific such as nausea and vomiting, which is usually followed by an asymptomatic period. Any child with a history of acute ingestion of greater than 200 mg/kg should be referred to a healthcare facility. If a toxic ingestion is suspected, a serum acetaminophen level should be calculated four hours after the reported time of ingestion. For patients who present to medical care more than four hours after ingestion, a stat acetaminophen level should be obtained. Acetaminophen levels obtained fewer than four hours after ingestion are difficult to interpret and cannot be used to estimate the potential for toxicity. The serum acetaminophen level is then plotted on the Rumack-Matthew nomogram and any level that is in the possible or probable hepatotoxicity range should then be treated with N-acetylcysteine.

A 16-year old boy presents with a superficial bite to his right arm. He states he was camping in the woods and a bat flew into his tent. Which of the following treatments should this patient receive?

Human rabies immunoglobulin and 4 doses of inactivated rabies vaccine over 14 days The rabies virus enters the CNS and causes a wide range of symptoms from headaches, anorexia, hallucinations, agitation to seizures, and hydrophobia. Bats are the most common animals to infect humans. Raccoons are the most common animal infected with rabies. The patient's history and presentation suggest an infection with the rabies virus and post-exposure prophylaxis with the inactivated rabies vaccine and the human rabies immunoglobulin should be administered. The human rabies immunoglobulin is a one time dose where as the inactivated rabies vaccine is given 4 times over 14 days. In addition the bite wound should be examined and cleaned and antibiotics may be of benefit.

A five-year-old boy is brought to the emergency room for altered mental status. The family had a party at their house the night before. In the morning, they found their son on the floor surrounded by empty cans of beer. He appears sleepy, and on the way to the hospital, he vomited twice. On examination, the boy is sedated with sluggish pupils and flushed skin. Which of the following abnormalities would you expect with the boy's toxic ingestion?

Hypoglycemia The signs and symptoms of the boy in the vignette are suspicious for ethanol ingestion. Children younger than six years of age often ingest ethanol when their exploratory behaviors lead them to unattended alcoholic beverages or to unsecured household products with high ethanol concentrations. Ethanol acts as a CNS sedative in a dose-dependent manner in overdose. Ethanol intoxication commonly manifests as altered behavior, lethargy, coma, ataxia, slurred speech, hypothermia, bradycardia, hypotension, and respiratory depression. A characteristic sickly sweet breath odor is often prominent. A history of ingestion from the caregiver or patient and characteristic physical findings are sufficient to establish the diagnosis of ethanol intoxication. Rapid blood glucose measurements should be performed in all patients with altered mental status after ethanol ingestion. Decreased blood glucose below 40 mg/dL is well described in children who have ingested ethanol. This ethanol-induced hypoglycemia results from exhausted glycogen reserves.

A 45-year-old man is brought to the emergency department after being the unrestrained passenger in a motor vehicle collision. His vital signs on arrival are T 37.3°C, HR 90, BP 140/94, RR 23. His physical exam is significant for a GCS of E3V4M5 and a large left temporal scalp laceration. CT imaging of his head reveals a left temporal linear skull fracture under the area of the laceration with no signs of intracranial hematoma formation. Which of the following therapies is indicated in this patient?

Intravenous antibiotics Linear skull fractures are single fractures that extend through the thickness of the calvarium. Linear skull fractures may or may not be associated with underlying trauma to the brain. Fractures that cross the path of the middle meningeal artery or a major venous sinus may cause significant bleeding. A linear fracture of the skull with an overlying scalp laceration is considered open. Patients with open skull fractures should receive antibiotic prophylaxis with vancomycin and ceftriaxone. This is also true of patients with skull fractures that are depressed, cross a sinus, or are associated with pneumocephalus.

Which of the following describes a patient with Brown-Séquard syndrome?

Ipsilateral loss of motor strength, vibratory sensation, and proprioception with contralateral loss of pain and temperature sensation below the level of injury Brown-Séquard syndrome results from hemitransection of the spinal cord with unilateral damage to the corticospinal and spinothalamic tracts. This injury is usually the result of penetrating injuries or a lateral mass fracture of the cervical spine. It is also caused by spinal cord tumors, infections, and ischemia. It is rarely seen in its pure form, typically occurring with incomplete involvement of related tracts. Prognosis is excellent, with most patients recovering.

A 20-year-old man presents to an urgent care clinic complaining of right foot pain after landing wrong while playing basketball. On physical exam, he has maximal tenderness over the lateral foot. A radiograph is obtained and shown above. Which of the following is the most likely diagnosis?

Jones fracture This man has a Jones fracture. A Jones fracture is a transverse fracture at the fifth metatarsal base, 1.5-3 cm distal to the proximal tuberosity. The most common mechanism of injury is significant adduction while the foot is plantar flexed. This mechanism may occur when an athlete lands awkwardly after jumping or when the ankle is inverted while running. Different portions of the proximal aspect of the fifth metatarsal have blood supply variations that contribute to differences in healing. The tuberosity receives blood supply from multiple metaphyseal arteries and from branches of the nutrient artery. The proximal diaphysis relies solely on the nutrient artery for blood supply, thereby increasing the likelihood for blood supply disruption and poor healing. Because most Jones fractures occur with ankle inversion, most patient present complaining of a sprained ankle. Applying the Ottawa ankle rules has been shown to detect the majority of Jones fractures, while reducing unnecessary radiographs. Three-view radiographs should be obtained in all patients meet the Ottawa rules. Initial treatment involves immobilization with a posterior splint, nonweightbearing status, and orthopedic referral in 3-5 days. Adjunctive therapies should include icing the area, elevation, and analgesics. Diaphyseal fractures often require surgical fixation.

Which of the following laboratory findings suggests ethylene glycol ingestion?

Lactate 0.9 mmol/L, pH 7.12, and creatinine 2.5 mg/dL The toxic alcohols include ethanol, ethylene glycol, methanol, and isopropanol. Each alcoholic overdose has its own clinical characteristic. Markers of intoxication with ethylene glycol include high anion gap metabolic acidosis with an absence of significant lactate or ketone concentrations, calcium oxalate formation leading to acute renal failure. Therefore, lactate 0.9 mmol/L, pH 7.12, and creatinine 2.5 mg/dL would be characteristic of ethylene glycol poisoning; although, lactate can be slightly elevated (both falsely or truly), but not at levels that would explain the the significant acidosis. Sodium bicarbonate may be used for urine alkalinization to improve acidemia. Ethanol or fomepizole may be utilized for acute management in conjunction with a nephrology consultation. Hemodialysis is indicated for ethylene glycol levels > 25 mg/dL. Adjunctive therapy includes thiamine and pyridoxine every 4 to 6 hours.

A two-year-old girl is brought to your clinic by her father because of abdominal pain. This is accompanied by decreased energy, vomiting, and constipation. CBC with peripheral smear shows microcytic anemia with basophilic stippling. Which of the following is the most likely diagnosis?

Lead poisoning The girl's symptoms and laboratory value are most likely due to lead poisoning. Since lead blocks iron from being incorporated into heme, lead poisoning can look like iron deficiency in which there is microcytic anemia. To differentiate this from iron deficiency, look for basophilic stippling of RBCs accompanied by gastrointestinal (GI) and central nervous system (CNS) symptoms. GI symptoms of lead poisoning include anorexia, abdominal pain, vomiting, and constipation, often occurring and recurring over a period of weeks. CNS symptoms are related to worsening cerebral edema and increased intracranial pressure. These symptoms include headaches, change in mentation, lethargy, papilledema, seizures, and coma. Lead lines are found on imaging of long bones. The most common pathway by which lead enters the body is through nonnutritive hand-to-mouth activity of young children. In most cases, lead is ingested, either as a component of dust licked off surfaces or in swallowed paint chips, through water contaminated by its flow through lead pipes or brass fixtures, or from food or liquids contaminated by contact with lead-glazed ceramic ware.

A 17-year-old man presents after being thrown a far distance off of a horse. Which of the following is consistent with an anterior cord syndrome?

Loss of pain and temperature, loss of motor function In order to fully understand the different syndromes of injuries to the spinal cord, it is imperative to understand the location of the tracts of the cord. The posterior columns carry tracts responsible for ipsilateral position and vibratory sensation. The lateral spinothalamic tract carries fibers for contralateral pain and temperature. The lateral corticospinal tract is responsible for ipsilateral motor function. Syndromes may be incomplete depending on how much of the cord is affected by the injury. In the anterior spinal cord syndrome, just the posterior columns are preserved and so patients lose all pain and temperature sensation as well as motor function. Most cases of anterior cord syndrome follow aortic surgery, but it has also been reported in the setting of hypotension, infection, vasospasm, or anterior spinal artery ischemia or infarct. In trauma, typically hyperflexion of the cervical spine causes the injury to the spinal cord.

Which of the following is the immediate first step in the treatment of chemical injuries to the eye?

Manual removal of particulate material followed immediately by irrigation with saline until the pH is 7.0 Chemical injuries to the eye represent one of the true ophthalmic emergencies. While almost any chemical can cause ocular irritation, serious damage generally results from either strongly basic (alkaline) compounds or acidic compounds. Alkali injuries are more common and can be more deleterious. Bilateral chemical exposure is especially devastating, often resulting in complete visual disability. Immediate, prolonged irrigation, followed by aggressive early management and close long-term monitoring, is essential to promote ocular surface healing and to provide the best opportunity for visual rehabilitation. Immediate irrigation with normal saline is the initial step even before complete exam when a patient presents with a chemical eye injury. A lid speculum should be placed and topical anesthesia applied. Irrigation may be administered by a handheld bottle or through IV tubing with an irrigation lens. The pH should be checked with a pH strip and irrigation discontinued when the pH reaches 7.0. Any particulate matter should be removed prior to irrigation if it is a reactive substance such as ammonium hydroxide crystals since fluid may dissolve these causing more injury. The upper lid should be everted to check for any particulate matter.

Which of the following drugs can cause mydriasis in overdose?

Meperidine At therapeutic doses, meperidine causes miosis (pinpoint pupils). However, unlike most other opioids, meperidine can cause mydriasis (dilated pupils) in cases of toxicity. Increased muscle tone, twitching, and tremors may also be seen with meperidine overdose. Meperidine is metabolized by the P450 system in the liver to the metabolite normeperidine, which can cause CNS toxicity at therapeutic meperidine doses. The half-life of normeperidine is up to 48 hours and subsequent hallucinations, seizures and psychosis may result. Therefore, meperidine is not routinely used for the management of acute pain in the Emergency Department. Meperidine can also potentially interact with monoamine oxidase inhibitors and cause serotonin syndrome.

A five-year-old boy is brought to the emergency department after being found unresponsive at home. He was found lying on the floor in his mother's room with prescription medications scattered all over. His mother called 911, and he was immediately rushed to the hospital by ambulance. On examination, the boy is sedated with a heart rate of 69 beats per minute, respiratory rate of 15 per minute, blood pressure 70/50 mm Hg, pulse oximetry of 99 percent, pupils 1-2 mm reactive to light, and 1+ reflexes on all extremities. Blood sugar is 200 and ECG shows QTc interval prolongation. Which of the following is the most likely medication ingested?

Methadone The boy has signs and symptoms consistent with opioid ingestion. Methadone ingestion can manifest with the classic opioid toxidrome of respiratory depression, sedation, and miosis. Signs of more severe toxicity can include bradycardia, hypotension, and hypothermia. Methadone has been associated with a prolonged QTc interval and risk of torsades de pointes. Patients with significant respiratory or CNS depression should be treated with naloxone, which is a mu receptor antagonist. Because the half-life of methadone is longer than naloxone, patients can require multiple doses of naloxone. Also, serial ECGs are needed to monitor for the development of a prolonged QTc interval. If a patient does develop a prolonged QTc, management includes close cardiac monitoring, repletion of electrolytes, and having magnesium readily available should the patient develop torsades de pointes.

You are called to the emergency department to examine a two-year-old girl for possible ingestion. She was accompanied by her grandparents who found her with an opened bottle of pills. There were two pills found in her mouth although the caretakers are not certain as to how many pills are missing. In transit to the ED, the girl was asymptomatic. Your examination at the ED reveals an alert girl with bradycardia, hypotension, normal pupillary size and reaction. You perform an ECG that shows sinus bradycardia. Blood glucose is normal. Which of the following medications could the girl most likely have ingested?

Metoprolol The findings on the girl are most consistent with beta-blocker toxicity like metoprolol. Beta-blockers competitively inhibit the action of catecholamines at the β receptor. Toxicity results in decreased chronotropy and inotropy in addition to slowing conduction through AV nodal tissue. These effects are manifested as bradycardia, hypotension, and heart block. Patients with reactive airways disease can experience bronchospasm due to blockade of β2-mediated bronchodilation. Beta-blockers interfere with glycogenolysis and gluconeogenesis, which can lead to hypoglycemia. Evaluation after an overdose should include an ECG and frequent reassessments of hemodynamic status. Blood glucose should be measured in all patients as hypoglycemia may be seen. Treatment involves supportive care and gastrointestinal decontamination as indicated. Glucagon and high-dose insulin are the the antidotes of choice for beta-blocker toxicity.

A three-year-old boy is brought to the Emergency Department by his father because of possible poisoning. He was found in the garage with furniture polish on his cheeks, mouth, and clothes. The father saw him coughing but denies choking or vomiting. On examination, the boy is active with normal vital signs. Which of the following is the next best step?

Obtain chest x-ray Hydrocarbon toxicity can result from ingestion of furniture polish. A transient and mild CNS depression is commonly noted after hydrocarbon ingestion or inhalation. Aspiration is characterized by coughing, which usually is the first clinical finding. It is important to obtain a chest radiograph. It may initially be normal, but they often show abnormalities within six hours of exposure in patients who have aspirated. Respiratory symptoms can remain mild or progress rapidly to the acute respiratory distress syndrome (ARDS) and respiratory failure. If hydrocarbon-induced pneumonitis develops, respiratory treatment is supportive. Corticosteroids or prophylactic antibiotics have not shown any clear benefit. Standard mechanical ventilation, high-frequency ventilation, and ECMO have all been used to manage the respiratory failure and ARDS associated with severe hydrocarbon-induced pneumonitis.

A five-year-old boy is playing with his two-year-old sister when he calls to tell his mother that the girl put a dime in her mouth. The mother could not find anything in the girl's mouth so she brings her to the emergency department. Your physical examination is normal. Which of the following is the next best step?

Obtain x-ray of the neck, chest, and abdomen Any history of foreign body ingestion should be taken seriously and investigated. The majority (80 percent) of foreign body ingestions occur in children between six months and three years of age. Coins and small toy items are the most commonly ingested foreign bodies. There may be symptoms of choking, gagging, and coughing that may be followed by excessive salivation, dysphagia, food refusal, emesis, or pain in the neck, throat, or sternal notch regions. There may be respiratory symptoms like stridor, wheezing, cyanosis, or dyspnea that may be encountered if the esophageal foreign body impinges on the larynx or the membranous posterior tracheal wall. Symptoms of cervical swelling, erythema, or subcutaneous crepitations suggest perforation of the oropharynx or proximal esophagus. The evaluation of a child with a history of foreign body ingestion should begin with plain anteroposterior (AP) radiographs of the neck, chest, and abdomen, along with lateral views of the neck and chest.

A five-year-old girl is rushed to the ED because of possible ingestion. She was unattended for a few minutes and later found playing with her grandmother's bag. The grandmother's bag contains over-the-counter medications and herbal products. The girl complained to her grandmother that her ears were buzzing. Upon arrival at the ED, the girl had one episode of vomiting. Her examination was normal except for diaphoretic skin. Which of the following is the medication that most likely caused her symptoms?

Oil of wintergreen The girl manifests signs and symptoms of salicylism. Oil of wintergreen contains 5 g of salicylate in one teaspoon and ingestion of very small volumes of this product has the potential to cause severe toxicity. Early signs of acute salicylism include nausea, vomiting, diaphoresis, and tinnitus. Moderate salicylate toxicity can manifest as tachypnea, tachycardia, and altered mental status. Signs of severe salicylate toxicity include hyperthermia, coma, and seizures. For the patient who presents soon after an acute ingestion, initial treatment should include gastric decontamination with activated charcoal. Initial therapy focuses on aggressive volume resuscitation and prompt initiation of sodium bicarbonate therapy in the symptomatic patient, even before obtaining serum salicylate levels. The primary mode of therapy is serum and urinary alkalinization wherein it enhances the elimination of salicylates by converting salicylate to its ionized form, "trapping" it in the renal tubules, and thus enhancing elimination.

A 45-year-old man presents to the emergency room after tripping over a curb. He is unable to put any weight on his right foot. On physical exam, there is bruising over the medial plantar surface and tenderness over the tarsometatarsal joint. Radiographs reveal an avulsion fracture of the second metatarsal and widening of the space between the medial cuneiform and base of the second metatarsal. Which of the following is the most appropriate treatment?

Open reduction and internal fixation Open reduction and internal fixation is indicated for Lisfranc injuries with any evidence of instability or bony fracture. A Lisfranc injury is characterized by a disruption the tarsometatarsal joints, which connect the forefoot to the midfoot. The injury can range from a mild sprain to severe dislocations with fracture. Lisfranc injuries are more common in males and in the third decade of life. The injury usually results from excessive indirect rotational forces and axial loading through a hyper-plantar flexed foot. Common causes of Lisfranc injuries include motor vehicle accidents, falls, or sports. Patients usually present with severe foot pain and an inability to bear weight. Physical exam may reveal midfoot bruising of the plantar surface, generalized swelling, and tenderness of the tarsometatarsal joint. Anteroposterior, lateral, and oblique radiographs are first line imaging. Stress radiographs may be necessary if non-weight bearing radiographs are unremarkable and there is high suspicion. Radiographic findings may include disruption of second metatarsal, avulsion fragments, or malalignment of the fourth metatarsal and the cuboid bone. Nonoperative management is indicated in patients with no displacement on stress radiographs and no evidence of bony injury. Operative management is indicated in patients with any evidence of instability or fracture. Posttraumatic arthritis is the most common complication of Lisfranc injuries.

You are called to examine a three-year-old boy in the emergency department for possible ingestion. He was found by his father drooling and playing with an opened drain cleaner. After suspecting a caustic ingestion, he immediately called poison control and was advised to bring the boy to the Emergency Department. The boy is irritable and drooling. His physical examination is normal. Which of the following is the most appropriate next step in management?

Order an upper endoscopy Caustic materials cause tissue injury. Caustic alkaline materials are found in drain cleaners, various cleaning agents, hair relaxers, dishwasher agents, and disk batteries. Alkalis produce liquefaction necrosis that allows further tissue penetration of the toxin and setting the stage for possible perforation. On the other hand, acids in household products include toilet bowl cleaners, swimming pool cleaners, and rust removers. Acids produce a coagulative necrosis that limits further tissue penetration, though perforation can still occur. Ingestion of caustic materials can produce injury to the oral mucosa, esophagus, and stomach. The symptoms include pain, drooling, vomiting, abdominal pain, and difficulty swallowing. Initial treatment of caustic exposures includes thorough removal of the product from the skin or eye by flushing with water. Endoscopy should then be performed within 12 to 24 hours of ingestion in symptomatic patients or those in whom injury is suspected.

You evaluate a 10-year-old girl in the ED because of headache. She has been complaining of headache, nausea, and dizziness. She was seen in clinic yesterday and was diagnosed with a viral illness. Her symptoms got worse, and she seems confused. The girl denies rhinorrhea, fever, loss of consciousness, trauma, or burns. Her mother and older brother also complain of headache. You obtain labs that show an elevated carboxyhemoglobin level. Which of the following is the most appropriate therapy?

Oxygen Carbon monoxide (CO) is the most common gas involved in pediatric exposures. Smoke inhalation is responsible for most inadvertent cases. CO is a colorless, odorless gas produced during the combustion of any carbon-containing fuel. Potential sources include wood-burning stoves, old furnaces, and automobiles. Early symptoms are nonspecific, including headache, malaise, nausea, and vomiting. At higher exposure levels, patients can develop mental status changes, confusion, ataxia, syncope, tachycardia, and tachypnea. Severe poisoning is manifested by coma, seizures, myocardial ischemia, acidosis, cardiovascular collapse, and potentially death. On exam, patients may have cherry-red skin. Evaluation should include a carboxyhemoglobin level in all symptomatic patients, arterial blood gas and creatine kinase in severely poisoned patients, and an ECG in any patient with cardiac symptoms. Treatment requires the administration of 100 percent oxygen to enhance elimination of CO. Severely poisoned patients might benefit from hyperbaric oxygen.

What is the most common pathogen found in dog bite wounds that should be taken into consideration when treating for infection?

Pasteurella Dog bites are the most common mammalian bite injuries worldwide. Infections from any mammalian bite are caused by organisms either from the oral flora of the biting mammal or from organisms found on human skin. Infection usually results from a mixture of organisms but some are more common than others in different animals. The predominant pathogen in dog bite injuries is Pasteurella and the choice of antibiotic should take this organism into consideration. Antibiotic prophylaxis is generally recommended for the following types of wounds: deep puncture wounds, moderate to severe wounds with crush injury, wounds in areas with underlying venous and lymphatic compromise, wounds on the hand, genitalia, face or close to a bone, wounds requiring closure, and wounds in immunocompromised hosts. First line antibiotic therapy for bite wounds is amoxicillin-clavulanate. If the patient has a penicillin allergy then either a fluoroquinolone plus clindamycin or metronidazole or trimethoprim/sulfamethoxazole plus clindamycin or metronidazole can be prescribed.

What is the most common pathogen found in infected cat bites?

Pasteurella multocida While only approximately 5% of dog bites become infected, a significant number of cat bites will become infected. Some studies have found up to 70% of cat bites become infected although this may be an overestimation given that many patients only seek treatment after an infection has developed. Cats have narrower, smaller teeth, resulting in most bites being deep puncture wounds. Cat bites are also more likely to be found on the hands which increases the risk of infection as well. Pathogens responsible for infection include those found in the oral flora of the cat as well as skin flora of the patient. The most common pathogen found in infected cat bites is Pasteurella multocida, a virulent facultatively anaerobic gram negative rod. Infection from Pasteurella typically occurs within 24 hours (earlier than cellulitis due to other pathogens) with erythema, warmth, swelling, and pain on examination. Purulent drainage is often seen as well. Prophylaxis with a 3-5 day course of amoxicillin-clavulanate should be prescribed to all patients who present with a cat bite. For those who present with wounds already infected, treatment options include ampicillin-sulbactam, a third-generation cephalosporin, or a fluoroquinolone.

A 34-year-old woman presents with elbow pain and diminished arm movement following a fall on her outstretched hand. Initial history and physical exam are notable for pain, swelling, and tenderness over the lateral elbow, and inability to fully extend the elbow. Which of the following is indicative of a radial head fracture?

Posterior fat pad sign Patients with radial head fractures typically present with localized swelling, tenderness, and decreased motion of the elbow following a fall on an outstretched hand. These fractures account for approximately 50% of elbow fractures in adults. They may be subtle and are easily missed on plain films. Presence of an elbow joint effusion, as indicated by elevation of the anterior or posterior fat pads or both, points to an occult intra-articular fracture. The posterior fat pad is not seen on a normal lateral X-ray of the elbow and thus is a more reliable indicator of joint effusion. The anterior fat pad can be visible as a thin translucent line on a normal elbow radiograph, but may bulge away from the joint in the setting of an effusion creating the anterior 'sail sign'. Tenderness of the lateral elbow combined with a positive fat pad sign indicate the injury should be treated as a radial head fracture.

A 30-year-old man presents to the emergency department after being involved in a head-on motor vehicle crash. He is complaining of severe pain in his right hip. On exam, his right leg appears to be shortened and is held slightly flexed, internally rotated, and adducted. Range of motion is severely limited due to pain. An anterior-posterior plain radiograph is negative for fractures, but the right femoral head appears smaller than the uninjured side. Which of the following is the most likely diagnosis?

Posterior hip dislocation This man most likely has a posterior hip dislocation. Posterior hip dislocations are most commonly associated with high-energy trauma such as motor vehicle collisions, falls from significant height, and high impact sports. Artificial hips can dislocate with less force. Posterior hip dislocations account for 90% of all hip dislocations. Posterior dislocations occur when a large axial load is transmitted through a flexed knee, which may occur when the knee comes into contact with the dashboard in a crash. Given the large amount of force needed to dislocate the femoral head, the majority of dislocations are associated with other injuries. Patients typically present with severe pain, an inability to bear weight, and deformity. Range of motion will be severely limited. A detailed neurovascular exam should be conducted to rule out sciatic nerve injury. In posterior dislocations, the affected leg tends to be held in slight flexion, adduction, and internal rotation. Plain radiographs are first-line imaging for diagnosing hip dislocations and associated fractures. A posteriorly dislocated femoral head will appear smaller than the contralateral side on anteroposterior film. After ruling out life-threatening injuries, the hip should be reduced within six hours. Concurrent femoral neck fracture is a contraindication for reduction. After reduction, computed tomography scan should be performed on all traumatic hip dislocation to evaluate for fractures. After reduction for simple dislocations, the patient should be on protected weight bearing for four to six weeks. Complications of hip dislocations include arthritis, femoral head osteonecrosis, sciatic nerve injury, and recurrent dislocations.

A 45-year-old man presents with left leg pain. He describes a sudden onset of pain in his calf area while playing basketball with his son. On exam, you squeeze his right calf and elicit plantar flexion of the right foot. When you do the same with his left calf there is no plantar flexion of his left foot. You administer ibuprofen and place the left leg in a posterior splint in plantar flexion. Which of the following is the most appropriate next step in management?

Refer to orthopedic surgery This patient has an abnormal Thompson test, a common clinical test used to diagnosed Achilles tendon rupture. Achilles tendon rupture typically occurs in the setting of sudden and forced dorsiflexion during athletic activity. Patients often report an audible "pop" and sudden pain in the calf area that improves or subsides quickly, followed by decreased ability to plantar flex the foot. The diagnosis is typically clinical. On physical examination, patients may exhibit a palpable tendon defect in the lower calf area. Some plantar flexion is usually maintained, even in cases of complete tendon rupture, due to the actions of the surrounding muscles. The Thompson test is a classic maneuver to assess the Achilles tendon. The patient lies prone with the legs flexed at the knee at 90 degrees. The examiner then squeezes the calf muscles and observes for passive plantar flexion of the foot. The injured extremity will have a weakened or no response compared to the uninjured side. Risk factors for Achilles tendon rupture include rheumatological diseases, chronic renal failure, steroid use, and recent treatment with fluoroquinolone antibiotics. Controversy remains regarding the best treatment. While early operative repair is associated with lower risk of rerupture and possibly improved functional outcome, it also has a higher rate of complications. Nonoperative management consists of a series of casts. Ultimately, the decision depends on the patient's age, activity level, and preferences, and is best made on a non-emergent basis in consultation with an orthopedic surgeon or sports medicine physician. Thus, initial management includes non-steroidal anti-inflammatory analgesics, crutches and non-weight-bearing status, immobilization of the ankle in plantar flexion with a splint, and outpatient referral.

Which of the following findings on electrocardiogram are indicative of left main coronary artery occlusion?

ST elevation in aVR > VI with horizontal ST depression in I, II and V4-6 By correctly interpreting an electrocardiogram, it is possible to determine the distribution of an acute myocardial infarction and the most likely coronary artery involved. ST segment elevation in lead aVR greater than 1 mm or greater than the elevation seen in the ST segment of V1 should prompt concern for occlusion of the left main coronary artery. The finding of ST elevation in aVR that is greater than that seen in VI can distinguish occlusion of the left main coronary artery from the left anterior descending artery with a sensitivity and specificity of approximately 80%. It was also found that the extent of ST segment elevation in aVR correlated with increased mortality. Other ECG findings consistent with left main occlusion include horizontal ST depression in leads I, II, and V4-6.

A 25-year-old man is found to have a tibia fracture after he was struck by a motor vehicle while crossing the road. An X-ray of his leg is seen above. What is the orientation of this fracture? Broken in two places

Segmented Characterizing tibia shaft fractures is most important when communicating with an orthopedic consultant. A single, large, free-floating segment of tibia bone between two well-defined fracture lines is a segmental fracture. In addition to orientation, fractures should also be described in terms of whether the fracture is open or closed, location of the fracture in the shaft, displacement and separation, and angulation. The fracture pattern will give a clue to the force that caused the injury. The tibia shaft requires a high degree of force to fracture and should prompt investigation for additional injuries. Tibia fractures are often open injuries because of the minimal amount of subcutaneous tissue between the fracture and the skin. Open fractures require immediate orthopedic consult and administration of IV antibiotics along with appropriate analgesia and splinting.

A 35-year old man presents to the emergency department with superficial facial lacerations and epistaxis. After treating the epistaxis, you examine the nasal mucosa. Which of the following is considered an emergency, requiring urgent treatment?

Septal hematoma A septal hematoma is considered an emergency. The problem is that the perichondrium, which supplies nutrition to the septum, is no longer in contact with the septum because of the intervening hematoma. Thus, the septal cartilage can necrose leading to a perforated septum. Septal hematomas should be drained acutely and the nose packed to keep the perichondrium in contact with the septal cartilage.

A 2-year-old girl is brought to the emergency room after she was found with an open bottle of her grandfather's pills. Mom states she had a couple episodes of vomiting before arrival. In the emergency room, she is tachypneic, febrile, and diaphoretic. Laboratory results reveal sodium 140, potassium 4.0, chloride 100, and bicarbonate 12. A venous blood gas shows pH 7.29, pCO2 22, and pO2 89. Which of the following is indicated in the management of this patient?

Sodium bicarbonate This patient presents with a mixed acid-base disorder due to salicylate toxicity from aspirin. Her labs are consistent with a metabolic acidosis and respiratory alkalosis. Salicylate overdose in children is not as common as it once was, since aspirin is no longer recommended for fever reduction in the this population. However, it can still occur when children have access to salicylate medications around the house. Presenting symptoms include tachypnea as the patient tries to compensate for the metabolic acidosis. Vomiting, diarrhea, fever, tinnitus, vertigo, diaphoresis, lethargy, and coma are also possible. Management is supportive, as with most toxicities, and should include alkalinizing the urine with sodium bicarbonate to aid in renal excretion. Severe cases may require dialysis.

A three-year-old girl was brought by her parents to the ED because of a possible ingestion. The girl was caught with an open bottle of amitriptyline although the mother is not sure how many tablets were missing. The girl did not experience any mental status changes, vomiting, abdominal pain, or fever. She arrived at the ED within 60 minutes of her possible ingestion. On examination, she is alert, active, with tachycardia, dry mucous membranes, and pupils 5-6 mm. Which of the following is the antidote for this type of ingestion?

Sodium bicarbonate The girl possibly took amitriptyline, which is a tricyclic antidepressant (TCA). Cardiovascular and CNS symptoms dominate the clinical presentation of TCA toxicity. Patients often develop features of the anticholinergic toxidrome such as delirium, mydriasis, dry mucous membranes, tachycardia, hyperthermia, mild hypertension, urinary retention, and slow GI motility. CNS toxicity can include lethargy, coma, myoclonic jerks, and seizures. Sinus tachycardia is the most common cardiovascular manifestation of toxicity. Patients can also develop widening of the QRS complex, premature ventricular contractions, and ventricular arrhythmias. Initial management should be directed to supporting vital functions. ECG should be obtained as soon as possible and followed serially to monitor for progression of toxicity. Sodium bicarbonate is the antidote of choice and works via overcoming the sodium channel blockade by providing a sodium load and via inducing an alkalosis to decrease drug binding to sodium channels. Indications for sodium bicarbonate include a QRS duration > 100 ms, ventricular dysrhythmias, and hypotension.

A 68-year-old man presents to the ED after a fall down twelve stairs. On physical exam, you note a large parietal scalp hematoma. His non-contrast computed tomography scan of the head is shown above. Which of the following is the most likely diagnosis?

Subdural hematoma This patient's imaging and physical examination are consistent with a traumatic acute subdural hematoma. A subdural hematoma is a collection of blood between the dura and the arachnoid mater. Subdural hematomas occur most commonly in individuals with brain atrophy, such as alcoholics and elderly patients. Subdural hematomas are significantly more common than epidural hematomas. Subdural hematomas occur when there is rupture of the bridging veins from movement of the brain relative to the skull, such as that seen with acceleration-deceleration injuries. This results in blood filling the potential space between the dura and arachnoid. Subdural hematomas may be acute, subacute, or chronic. Acute subdural hematomas are characteristically seen after a traumatic head injury. Patients typically present with headache, mental status changes, seizures, or focal deficits. Subacute or chronic subdural hematomas may occur without reported history of trauma in up to 50% of patients. These patients may present with symptoms similar to those seen in acute subdural hematomas; however, they may also present with more indolent symptoms such as personality changes, depression, or memory deficits. Subdural hematoma is diagnosed on non-contrast CT of the head and appears as a crescent-shaped hematoma that may cross suture lines. Management of subdural hematomas includes neurosurgical consultation for possible surgical evacuation; however, small subdural hematomas may be closely observed with serial non-contrast CTs of the head. Indications for emergent surgery in a patient with a subdural hematoma include neurologic deterioration or > 5 mm midline shift on CT.

A 37-year-old man is brought into the emergency department after being stabbed in the flank during a bar fight. On examination, the patient has 2/5 strength and decreased vibration sense in his left lower extremity as well as decreased pinprick sensation in his right lower extremity. Which of the following is the most likely pattern of this patient's injury?

The spinothalamic tract was transected on the left Brown-Séquard syndrome is a hemicord syndrome which manifests as ipsilateral loss of motor function, vibration, and proprioception and contralateral loss of pain and temperature sensation. Brown-Séquard syndrome most commonly occurs due to penetrating trauma to the spine but can less commonly result from lateral cord compression due to spine fractures, hematomas, herniated discs, or tumors. Loss of motor function occurs due to transection or compression of the ipsilateral corticospinal tract, which carries upper motor neurons from the motor cortex of the brain. The same is true of the ipsilateral dorsal column, which carries sensory neurons for proprioception and vibration to the sensory cortex. In contrast, the spinothalamic tract, which carries pain and temperature sensation, travels two levels along the spinal tract before decussating (crossing over), resulting in loss of pain and temperature sensation contralateral to the lesion. Thus, in Brown-Séquard syndrome, the spinothalamic tract is always damaged contralateral to the side of pain and temperature loss, in this case, on the left.

Which of the following distinguishes traumatic injuries of the thoracic spine from injuries to the remainder of the spinal column?

Thoracic spine injuries are more likely to be associated with cord injuries The thoracic spine is more rigid than other segments of the spine and its relative stiffness is furthered by articulation with the ribs. Due to its rigidity, the thoracic spine is less commonly injured than other spinal segment and requires a more severe traumatic force to cause injury. As such, thoracic spine injuries are commonly associated with intrathoracic injuries due to the severity of the traumatic force required to injure the thoracic vertebrae. Furthermore, the spinal canal in the thoracic spine is more narrow than in the cervical and lumbar spine, increasing the rate of spinal cord injury in trauma of the thoracic spine. Spinal cord injuries associated with thoracic spine injuries are more likely to be complete cord lesions than spinal cord injuries associated with cervical, lumbar, or transitional zone trauma.

A 14-year-old girl presents to the ED with wrist pain one day after falling onto an outstretched hand while playing soccer. She has normal sensation and movement and a normal radial pulse. Her exam is unremarkable with the exception of tenderness at the anatomical snuffbox. X-rays of her wrist and forearm are normal. Which of the following is the next best step in management?

Thumb spica splint Scaphoid fractures are relatively uncommon in children but are at risk of nonunion, can cause significant long-term pain, and may have normal initial X-rays. The arterial supply of the scaphoid bone enters distally leading to decreased blood and nutrient supply proximally. A fracture through the proximal or waist (central ⅓) of the bone can decrease blood supply further, especially if the artery is disrupted. This causes poor healing conditions which can lead to nonunion or osteonecrosis. In addition, many scaphoid fractures are extremely difficult to visualize on X-ray and swelling is not always present on exam. Patients with a scaphoid fracture may only have pain at the anatomical snuffbox after a fall onto an outstretched hand. Current recommendations are to place such patients in a thumb spica splint and refer to outpatient orthopedics to determine the need for further immobilization and repeat X-rays in two weeks.

Which of the following bone fractures is most commonly involved in acute compartment syndrome?

Tibia Muscles in the human body are divided into compartments lined with fascial membranes. Acute compartment syndrome (ACS) occurs when increased tissue pressure within a compartment results in nerve and muscle ischemia. ACS generally develops after a significant trauma such as a fracture, but may also occur with minor trauma or due to a nontraumatic etiology. The most common sites for ACS to occur are the leg and arm. Long bone fractures are the cause of 75% of cases of ACS, with the tibia being the most common bone involved. ACS is considered to be a surgical emergency. Measurement of compartment pressures assists in determination of the need for fasciotomy, which is the definitive treatment for ACS. Frequent, serial examinations are critical in the management of patients at risk for ACS. Prognosis is good in patients with early diagnosis and appropriate intervention.

A ten-year-old boy collapses on the football field and is quickly evaluated by the sports medicine doctor. He had been practicing on the field for approximately two hours, without any rest. His temperature is 105°F and his skin is sweaty. On testing his mental status, he can say his name and age but does not know where he is. Which of the following is the most appropriate initial treatment for this boy?

Whole body cold-water immersion Heat stroke is a severe illness with mortality rates of up to 50%. It is characterized by CNS abnormalities and may lead to multi-organ damage. It may occur in the setting of intense physical activity during sports play. Rectal temperature is usually > 104°F, and patients will have profuse sweating. In contrast, "classic" heat stroke is seen in elderly patients and is usually a slower onset, with the physical exam notable for dry, hot skin. Treatment of children with heat stroke consists of immediate whole body cooling, usually performed by cold water immersion. Rapid cooling should be continued until the temperature decrease to 101 or 102°F. Careful monitoring of the airway, breathing, circulation, temperature, and CNS status is crucial. IV fluids should also be initiated as soon as possible, at a rate of 800 ml/m2 in the first hour.

A 23-year-old man presents to the Emergency Department after closing his right hand in a car door. He is complaining of 10/10 sharp pain to his right index finger. On examination, you note the distal phalanx of the right index finger has ulnar deviation. It is edematous and ecchymotic. The fingernail is intact, however, a subungual hematoma occupying approximately 25% of the nail is present. What is the first step in the management of the subungual hematoma?

X-ray of the right index finger Subungual hematomas are commonly seen in primary care and in the emergency room following simple trauma resulting in bleeding into the the space between the nail bed and the fingernail. The pain associated with this condition is caused by the pressure created by the blood occupying an enclosed space. Trephination of the fingernail will relieve the pain associated with this condition, however, is not contraindicated if there is a suspected fracture of the distal phalanx. It is contraindicated if there is suspected subungual melanoma or if the nail is crushed or fractured. Because of the patient's history of a crush injury and clinically observed ulnar deviation of the distal phalanx, an X-ray of the right index finger is the first step in the management of this subungual hematoma.

Which of the following is most likely to be associated with a bilateral interfacetal dislocation?

complete cord transection Bilateral facet dislocation is an unstable injury that occurs from forceful hyperflexion of the neck. It occurs when the articular masses of one vertebra dislocate anteriorly and superiorly from the articular surface of the vertebra below it causing anterior displacement of the spine. This injury involves forceful disruption of multiple structures at the level of the injury, including all ligamentous structures, the articular facet joints, and the intervertebral disc. This is commonly associated with a complete spinal cord injury due to transection of the cord at the level of the injury. Diagnosis is made by radiographic evidence of displacement of the superior vertebral body anteriorly more than one half of its width.

A 45-year-old chemist presents to the emergency department after accidentally spilling elemental aluminum on his left hand and forearm 30 minutes prior to arrival. What is the most appropriate next step?

cover the affected area with mineral oil The first step in managing elemental metal burns is to covering the affected area in mineral oil, sand, or foam from a Class D fire extinguisher. Chemical burns differ from thermal burns, in that chemical burns can continue to cause damage at long as the active chemical is in contact with the skin and can cause systemic toxicity if absorbed or inhaled. Chemical burns also heal more slowly and require longer hospitalization periods. When encountering a chemical burn, it is critical that treatment be started as soon as possible. General steps for management of topical chemical burns includes securing the safety of rescuers and health care works, removing the patient from the area of exposure, removing the patient's clothing and jewelry, and brushing any dry chemicals off the patient. Unless use of water is contraindicated, copious irrigation with moderately warm water at low pressures is essential for preventing further tissue damage. Chemicals that should not be immediately irrigated with water include dry lime, elemental metals (e.g. sodium, potassium, magnesium, and phosphorus), and phenols. Treatment for elemental metal burns involves removal of the substance and the affected area covered in mineral oil to prevent further exposure to air and moisture. Surgical debridement may be required if fragments are embedded in the wound. After decontamination, chemical burn patients are managed similar to thermal burn patients with fluid resuscitation and prevention of further complications like rhabdomyolysis, infection, and hypothermia. Medical toxicology and poison control consultations should be made in most cases of chemical burns.

A 19-year-old migrant worker presents to the ED with vomiting, diarrhea, diaphoresis, wheezing, and excessive tearing. Vital signs are BP 150/100 mm Hg, HR 36 beats per minute, RR 28 breaths per minute, and T 98.6°F. Which of the following is the most appropriate initial step in management?

decontamination This patient is exhibiting signs and symptoms consistent with cholinergic toxicity secondary to organophosphate poisoning. Management of organophosphate poisoning should begin with decontamination. Anyone coming into contact with the patient should wear a gown and gloves to prevent transmission of the toxin. Clothing should be removed and discarded in a well-ventilated area and the patient should be thoroughly washed including ocular irrigation. The patient should then be resuscitated in a similar manner to any other ED patient. These patients often require early intubation and aggressive IV fluid resuscitation. There are two keys to definitive treatment in patients with organophosphate poisoning. The first is to temporize the life-threatening signs and symptoms of cholinergic toxicity. Atropine is a competitive inhibitor of acetylcholine at muscarinic receptors. The goal of treatment with atropine is to titrate to the drying of bronchial secretions. Pralidoxime (2-PAM) is the definitive antidote to organophosphate poisoning. 2-PAM forms a complex with the bound acetylcholinesterase enzyme to cause the release of the organophosphate from the enzyme. This results in regeneration of its ability to metabolize acetylcholine. Organophosphates bind tightly to acetylcholinesterase preventing the breakdown of acetylcholine. Signs and symptoms of organophosphate poisoning relate to excess acetylcholine at the nicotinic and muscarinic receptors. This produces the cholinergic toxidrome due to high postsynaptic parasympathetic activity, resulting in the classic SLUDGE mnemonic: Salivation, Lacrimation, Urination, Diarrhea, GI cramps and Emesis. Patients will also demonstrate diaphoresis, pupillary miosis, bradycardia, muscular fasciculations, paralysis, agitation, seizures, or even coma.

A two-year-old boy was brought to the emergency department because of vomiting. About 30 minutes prior, he was found by his parents with an open bottle containing ferrous fumarate tablets. The mother estimates about five tablets are missing, and each tablet contains 65 mg of elemental iron. The boy had three episodes of non-bloody vomiting. The physical examination is essentially normal except for tachycardia. Which of the following is the next best step?

get serum iron level four hours after ingestion The patient has symptoms and a history compatible with iron toxicity. The severity of an exposure is related to the amount of elemental iron ingested. Ferrous sulfate contains 20 percent elemental iron, ferrous gluconate has 12 percent, and ferrous fumarate contains 33 percent. Iron is directly corrosive to the GI mucosa, which can lead to hematemesis, melena, ulceration, infarction, and potential perforation. For significant ingestions (> 20 mg/kg of elemental iron), especially when tablets are identified on the abdominal radiograph, whole-bowel irrigation with a polyethylene glycol electrolyte lavage solution (PEG-ELS) is routinely recommended. Iron toxicity is described in four stages. The initial stage, 30 minutes to six hours after ingestion consists of profuse vomiting and diarrhea, abdominal pain, and significant volume losses, leading to potential hypovolemic shock. The second stage, six to 24 hours after ingestion, is the quiescent phase where GI symptoms typically resolve. In the third stage, occurring 12 to 24 hours after ingestion, patients develop multi-system organ failure, shock, hepatic and cardiac dysfunction, acute lung injury, and profound metabolic acidosis. Symptomatic patients and patients with a large exposure by history should have serum iron levels drawn four to six hours after ingestion. Serum iron concentrations of < 500 µg/dL four to eight hours after ingestion suggest a low risk of significant toxicity, whereas concentrations of > 500 µg/dL indicate significant toxicity.

A 22-year-old woman presents to the emergency department after developing a widespread rash following a bee sting. The patient reports she has been stung once previously but never had a reaction. She denies difficulty breathing but states she feels somewhat light-headed. She is otherwise healthy, and has no significant past medical history. Vital signs are T 37, BP 85/60, HR 100, RR 18, oxygen saturation is 98%. A diffuse urticarial rash is present on the patient's extremities. Which of the following is the next best step in management?

intramuscular epinephrine This woman's presentation is concerning for anaphylaxis. Anaphylaxis is characterized by upper airway obstruction, rash, bronchospasm, and hypotension or cardiovascular collapse. Although she does not yet have trouble breathing, epinephrine is indicated. Anaphylaxis occurs as a result of IgE-mediated hypersensitivity that causes mast cell degranulation and histamine release. Patients typically present with a combination of hives, facial edema, pruritus, respiratory difficulty, and hypotension in the setting of an inciting factor such as bee sting, peanuts, shellfish and other foods. Anaphylaxis generally does not occur during the patient's first exposure to the allergen, instead presenting following the subsequent exposures. Intramuscular epinephrine and the close monitoring of vital signs are the foundation of anaphylaxis treatment. Patients with a history of reactive airway disease may benefit from albuterol nebulizers as a supplement. Intramuscular epinephrine should be administered every 5-15 minutes until the symptoms resolve.


Conjuntos de estudio relacionados

Immunology Exam 1 Practice Questions

View Set

AP World History - Chapter 14 - The Spread of Chinese Civilization: Japan, Korea, and Vietnam

View Set

Ultimate Vocabulary (Class of 2026) through 4th Quarter List II

View Set

Intro to Business: Strategic Management

View Set